Autor Tema: UTF sin descenso

0 Usuarios y 1 Visitante están viendo este tema.

26 Febrero, 2024, 05:36 pm
Leído 104 veces

Fernando Moreno

  • $$\Large \color{#5e8d56}\pi\,\pi\,\pi$$
  • Mensajes: 488
  • País: es
  • Karma: +1/-0
  • Sexo: Masculino
Hola,     


Supongamos que  \( a^3+b^3+c^3=0 \) ,  para  \( a,b,c \)  enteros -y- coprimos entre sí.    

Si  \( 3 \)  no divide á  \( abc \) ;  puesto que  \( (\mathbb{Z}/9\mathbb{Z})^3=\{0,1,-1\} \) ,  tendremos que  \( a^3+b^3+c^3\not\equiv{0} \) mod \( 9 \) .  Lo que no puede ser.      

Luego  \( 3 \)  debe dividir á  \( abc \) .  Y no perdemos generalidad si suponemos que  \( 3^k \) ,  para  \( k\in{\mathbb{N^+}} \) ,  divide á  \( c \) .      

Entonces:   

\( -c^3=a^3+b^3=(a+b)((a+b)^2-3ab) \) .  Donde  \( a+b \)  -y-  \( (a+b)^2-3ab \)  serán coprimos y terceras potencias salvo por  \( 3 \) ;  puesto que  \( 3 \) ,  que divide á  \( c \) ,  debe dividir á  \( a+b \) .  Así:  \( 3^{3k-1} \)  dividirá á  \( a+b \)  -y- sólo  \( 3 \)  á  \( (a+b)^2-3ab \) .   

Además, ocurre que:  \( (a+b)^3=a^3+b^3+3ab(a+b) \)  \( \Rightarrow \)  \( (a+b)^3=-c^3+3ab(a+b) \)  -y-  \( (a+b)^3=3ab(a+b)-3^{3k}c'\,^3 \) .    

En  \( \mathbb{Z} \)  existe el número  \( a+b-3^k \) , al que llamaremos  \( q \) .  Luego  \( a+b\equiv 3^k \) mod \( q \) .    

De esta manera, módulo  \( q \) , será que  \( (a+b)^3\equiv 3ab(a+b)-3^{3k}c'\,^3 \)  -y- :  \( 3^{3k}\equiv 3^{k+1}ab-3^{3k}c'\,^3 \) .  Luego  \( 3^{3k}\equiv 3^{k+1}(ab-3^{2k-1}c'\,^3) \) mod \( q \) .  Y por consiguiente:  \( ab-3^{2k-1}c'\,^3\equiv 3^{2k-1} \) mod \( q \) .      

Ahora bien, como  \( 3 \)  es un factor primo de  \( q \) ,  debe cumplirse también que  \( ab-3^{2k-1}c'\,^3\equiv 3^{2k-1} \) mod \( 3 \) .  Pero:  \( ab-3^{2k-1}c'\,^3-3^{2k-1}\not\equiv 0 \) mod \( 3 \) .          


La generalización es rápida, pero no vale la pena detenerse en ella si lo anterior ya está mal. De forma esquemática, voy a plantearla cómo sería en el caso del UTF5 para   \( a^5+b^5+c^5=0 \)  -y-  \( c \)  múltiplo de  \( 5^k \) .    

Tenemos:   

\( (a+b)^5=a^5+5a^4b+10a^3b^2+10a^2b^3+5ab^4+b^5 \) 

\( (a+b)^5=-c^5+5ab(a^3+2a^2b+2ab^2+b^3) \) 

\( (a+b)^5=5ab(a^3+b^3+2ab(a+b))-c^5 \) 

\( (a+b)^5=5ab((a+b)(a^2+b^2-ab)+2ab(a+b))-5^{5k}c'\,^5 \) 

\( (a+b)^5=5ab(a+b)(a^2+b^2+ab)-5^{5k}c'\,^5 \) 

\( (a+b)^5=5ab(a+b)((a+b)^2-ab)-5^{5k}c'\,^5 \) 

\( (a+b)^5=5ab(a+b)^3-5a^2b^2(a+b)-5^{5k}c'\,^5 \) 

En  \( \mathbb{Z} \)  existe  el número  \( a+b-5^k \) ,  al que llamaremos  \( q \) .  Donde:  \( a+b\equiv 5^k \) mod \( q \) .    

Luego módulo  \( q \) ,  tendremos: 

\( (a+b)^5\equiv 5ab(a+b)^3-5a^2b^2(a+b)-5^{5k}c'\,^5 \) 

\( 5^{5k}\equiv 5^{3k+1}ab-5^{k+1}a^2b^2-5^{5k}c'\,^5 \) 

\( 5^{5k}\equiv 5^{k+1}(5^{2k}ab-a^2b^2-5^{4k-1}c'\,^5) \) 

Por tanto:  \( 5^{2k}ab-a^2b^2-5^{4k-1}c'\,^5\equiv 5^{4k-1} \) mod \( q \) .    

Ahora bien,   \( 5 \)  es un factor primo de  \( q \) ,  porque puede probarse que divide á  \( a+b \) ; pero:  \( 5^{2k}ab-a^2b^2-5^{4k-1}c'\,^5-5^{4k-1}\not\equiv 0 \) mod \( 5 \) .     


Si todo esto fuera correcto, ésta sería sin duda la demostración que tenía Pierre de Fermat.   


Un saludo,
An expert is a man who has made all the mistakes, which can be made, in a very narrow field. Niels Bohr

27 Febrero, 2024, 10:20 am
Respuesta #1

Luis Fuentes

  • el_manco
  • Administrador
  • Mensajes: 56,141
  • País: es
  • Karma: +0/-0
Hola

De esta manera, módulo  \( q \) , será que  \( (a+b)^3\equiv 3ab(a+b)-3^{3k}c'\,^3 \)  -y- :  \( 3^{3k}\equiv 3^{k+1}ab-3^{3k}c'\,^3 \) .  Luego  \( 3^{3k}\equiv 3^{k+1}(ab-3^{2k-1}c'\,^3) \) mod \( q \) .  Y por consiguiente:  \( ab-3^{2k-1}c'\,^3\equiv 3^{2k-1} \) mod \( q \) .      

No es cierto que si \( a\cdot b\equiv a\cdot c \) mod \( q \) entonces \( b\equiv c \) mod \( q \).

Spoiler
Para poder garantizar esa implicación necesitarías que \( a \) fuese inversible mod \( q \), o lo que es lo mismo, que \( a,q \) fuesen coprimos.
[cerrar]

Por ejemplo \( 2\equiv 18 \) mod \( 16 \) pero \( 1\not\equiv 9 \) mod \( 16 \)

Saludos.

27 Febrero, 2024, 10:52 pm
Respuesta #2

Fernando Moreno

  • $$\Large \color{#5e8d56}\pi\,\pi\,\pi$$
  • Mensajes: 488
  • País: es
  • Karma: +1/-0
  • Sexo: Masculino
Hola Luis. Gracias por la revisión. Me tienes que disculpar, pero no acabo de ver completamente lo que dices.

Por ejemplo \( 2\equiv 18 \) mod \( 16 \) pero \( 1\not\equiv 9 \) mod \( 16 \)

Te pongo yo un ejemplo sobre tu ejemplo:  \( 2^2\equiv 2^2\cdot 9^2 \) mod \( 16 \)  -y- sí que cumple que  \( 9^2\equiv 1 \) mod \( 16 \) .  Y es claro que módulo  \( q \) :  \( (3^{3k})^s\equiv (3^{k+1}ab-3^{3k}c'\,^3)^s \)  -y- que  \( (3^{2k-1})^s\not\equiv(ab-3^{2k-1}c'\,^3)^s \) mod \( 3 \) ,  para un  \( s \)  entero.

Yo el ejemplo que he tenido todo el rato en la cabeza es el de módulo  \( 15=3\cdot 5 \) .  Si  \( 18\equiv 3 \) mod \( 15 \) ,  entonces:  \( 18\equiv 3 \) mod \( 3 \)  -y-  \( 18\equiv 3 \) mod \( 5 \) .

Tenemos que  \( q=a+b-3^k=3^{3k-1}c_1^3-3^k \) ,  para un  \( c_1 \)  componente de  \( c=3^kc_1c_2 \) .  Luego  \( q=3^k(3^{2k-1}c_1^3-1) \) ,  donde  \( 3^k \)  -y-  \( 3^{2k-1}c_1^3-1 \)  son coprimos, parecidos al caso  \( 3\cdot 5=15 \) .

De esta manera  \( ab-3^{2k-1}c'\,^3 \)  sí es coprimo módulo  \( 3^k \)  componente de  \( q \) . 

Yo no estoy planteando el caso general, que es como dices, sino un caso particular dentro del mismo, que como ves admite un desarrollo.

Un saludo
An expert is a man who has made all the mistakes, which can be made, in a very narrow field. Niels Bohr

28 Febrero, 2024, 10:20 am
Respuesta #3

Luis Fuentes

  • el_manco
  • Administrador
  • Mensajes: 56,141
  • País: es
  • Karma: +0/-0
Hola

Hola Luis. Gracias por la revisión. Me tienes que disculpar, pero no acabo de ver completamente lo que dices.

Una reflexión previa.

- Un sólo ejemplo sirve para justificar que una determinada propiedad NO se cumple en general. Eso tira abajo una demostración donde se haya usado esa propiedad. Si se piensa que en realidad lo que se ha usado es una propiedad más particular, hay que decir cual y DEMOSTRARLA.
- Un ejemplo a favor NO vale para determinar que una propiedad es CIERTA. Por ejemplo \( 2+2=2\cdot 2 \), pero estaríamos muy errados si pretendiésemos con eso decir nada a favor de que en general \( a+a=a\cdot a \).

Citar
Te pongo yo un ejemplo sobre tu ejemplo:  \( 2^2\equiv 2^2\cdot 9^2 \) mod \( 16 \)  -y- sí que cumple que  \( 9^2\equiv 1 \) mod \( 16 \) .


 Por lo que te comento en esa reflexión previa: ese ejemplo no me dice nada.

Citar
Y es claro que módulo  \( q \) :  \( (3^{3k})^s\equiv (3^{k+1}ab-3^{3k}c'\,^3)^s \)  -y- que  \( (3^{2k-1})^s\not\equiv(ab-3^{2k-1}c'\,^3)^s \) mod \( 3 \) ,  para un  \( s \)  entero.

 ¿y bien...?

Citar
Yo el ejemplo que he tenido todo el rato en la cabeza es el de módulo  \( 15=3\cdot 5 \) .  Si  \( 18\equiv 3 \) mod \( 15 \) ,  entonces:  \( 18\equiv 3 \) mod \( 3 \)  -y-  \( 18\equiv 3 \) mod \( 5 \) .

Otra vez: un ejemplo no dice nada. Te puede ayudar a sospechar que una determinada propiedad general puede ser cierta bajo determinadas condiciones. Pero llegado el momento tienes que indicar exactamente que propiedad, que condiciones y demostrarla.

Ni siquiera veo claro una analogía con el caso que nos ocupa.

Citar
Tenemos que  \( q=a+b-3^k=3^{3k-1}c_1^3-3^k \) ,  para un  \( c_1 \)  componente de  \( c=3^kc_1c_2 \) .  Luego  \( q=3^k(3^{2k-1}c_1^3-1) \) ,  donde  \( 3^k \)  -y-  \( 3^{2k-1}c_1^3-1 \)  son coprimos, parecidos al caso  \( 3\cdot 5=15 \) .

De esta manera  \( ab-3^{2k-1}c'\,^3 \)  sí es coprimo módulo  \( 3^k \)  componente de  \( q \) . 

Pero eso no tiene nada que ver con lo que digo.

Lo que digo es tu has pasado de aquí:

\( 3^{3k}\equiv 3^{k+1}(ab-3^{2k-1}c'\,^3)\quad \mod\quad q \)

aquí:

\( 3^{2k-1}\equiv (ab-3^{2k-1}c'\,^3)\quad \mod\quad q \)

y tienes que justificar claramente ese paso. Aparentemente lo que haces es dividir a ambos lados por \( 3^{k+1} \) y te digo que eso no puede hacerse alegremente en aritmética modular; sólo estaría garantizado si \( 3^{k+1} \) fuese coprimo con \( q \) cosa que NO es cierta.

Afinando más tu dices ahora que \( q=3^k(3^{2k-1}c_1^3-1)=3^k\cdot M \) donde \( M \) no es múltiplo de \( 3 \).

Entonces:

\( 3^{3k}\equiv 3^{k+1}(ab-3^{2k-1}c'\,^3)\quad \mod\quad q \)

significa que:

\( 3^{3k}=3^{k+1}(ab-3^{2k-1}c'\,^3)+3^k\cdot M\cdot T \)

de donde \( T \) es divisible por \( 3 \); y dividiendo por \( 3^{k+1} \) te quedaría:

\( 3^{2k-1}=(ab-3^{2k-1}c'\,^3)+M\cdot (T/3) \)

Así que lo que puedes afirmar es que:

\( 3^{2k-1}\equiv (ab-3^{2k-1}c'\,^3)\quad \mod\quad M \) (pero no módulo \( q \)).

Saludos.

28 Febrero, 2024, 05:39 pm
Respuesta #4

Fernando Moreno

  • $$\Large \color{#5e8d56}\pi\,\pi\,\pi$$
  • Mensajes: 488
  • País: es
  • Karma: +1/-0
  • Sexo: Masculino
Hola Luis, llevas razón, voy a intentar demostrar lo que hago.

Lo que digo es tu has pasado de aquí:

\( 3^{3k}\equiv 3^{k+1}(ab-3^{2k-1}c'\,^3)\quad \mod\quad q \)

aquí:

\( 3^{2k-1}\equiv (ab-3^{2k-1}c'\,^3)\quad \mod\quad q \)

Esto:  \( 3^{3k}\equiv 3^{k+1}(ab-3^{2k-1}c'\,^3) \) mod \( q \) ,  es equivalente a esto:  \( 27\equiv 9\cdot x \) mod \( (q=3^k(3^{2k-1}c_1^3-1)) \) ,  si  \( k=1 \) .  Y por tanto equivalente a esto:  \( 27\equiv 9\cdot x \) mod \( (3\cdot y) \) .  Imaginemos que  \( 3\cdot y \)  es menor que  \( 27 \) .  Entonces, como dices, efectivamente  \( x \)  no tiene porqué ser a la fuerza congruente con  \( 3 \)  módulo  \( (3\cdot y) \) .  Supongamos que  \( y=2 \) .  Será que  \( 27\equiv 9\cdot x \) mod \( 6 \)  \( \Rightarrow{} \)  \( 3\equiv 9\cdot x \) mod \( 6 \)  -y-  \( x\equiv 1 \) mod \( 6 \) .

Ahora bien, si  \( (3\cdot y) \)  fuera mayor que  \( 3^{3k} \) ,  entonces por fuerza  \( ab-3^{2k-1}c'\,^3 \)  debe ser congruente con  \( 3^{2k-1} \) .  Es como si en la equivalencia de antes digo que  \( y=13 \)  -y-:  \( 27\equiv 9\cdot x \) mod \( 39 \) .  Debe ser  \( x\equiv 3 \) mod \( 39 \) .  ¿Estás de acuerdo?

Te demuestro ahora que estamos en este último caso. Tenemos que  \( q=3^k(3^{2k-1}c_1^3-1) \) .  Sabemos que como mínimo  \( k=2 \)  -y- que si  \( 3 \)  divide á  \( c \) ,  \( 2 \)  también lo divide; en concreto que divide á  \( c_1 \) .  Se puede demostrar, lo dejo ahora en el aire. Por tanto:  \( 3^{3k}=729 \)  -y-  \( q=3^k(3^{2k-1}c_1^3-1)=9\cdot(27\cdot 8-1)=1935 \) ,  en el caso mínimo que nada más  \( c_1=2 \) .  Entonces  \( 3^{3k}\equiv 3^{k+1}(ab-3^{2k-1}c'\,^3) \) mod \( q \)  \( \Rightarrow{} \)  \( 729\equiv 27(ab-3^{2k-1}c'\,^3) \) mod \( 1935 \)  \( \Rightarrow{} \)  \( ab-3^{2k-1}c'\,^3\equiv 27 \) mod \( 1935 \) .

Saludos
An expert is a man who has made all the mistakes, which can be made, in a very narrow field. Niels Bohr

28 Febrero, 2024, 07:43 pm
Respuesta #5

Luis Fuentes

  • el_manco
  • Administrador
  • Mensajes: 56,141
  • País: es
  • Karma: +0/-0
Hola

Ahora bien, si  \( (3\cdot y) \)  fuera mayor que  \( 3^{3k} \) ,  entonces por fuerza  \( ab-3^{2k-1}c'\,^3 \)  debe ser congruente con  \( 3^{2k-1} \) .  Es como si en la equivalencia de antes digo que  \( y=13 \)  -y-:  \( 27\equiv 9\cdot x \) mod \( 39 \) .  Debe ser  \( x\equiv 3 \) mod \( 39 \) .  ¿Estás de acuerdo?

No. \( 9\cdot 16=27 \) mod \( 39 \), por ejemplo. Creo que deberías de analizar la última parte de mi última respuesta. Te dice exactamente como interpretar la relación que tienes y que se supone que puedes deducir de ella:

Afinando más tu dices ahora que \( q=3^k(3^{2k-1}c_1^3-1)=3^k\cdot M \) donde \( M \) no es múltiplo de \( 3 \).

Entonces:

\( 3^{3k}\equiv 3^{k+1}(ab-3^{2k-1}c'\,^3)\quad \mod\quad q \)

significa que:

\( 3^{3k}=3^{k+1}(ab-3^{2k-1}c'\,^3)+3^k\cdot M\cdot T \)

de donde \( T \) es divisible por \( 3 \); y dividiendo por \( 3^{k+1} \) te quedaría:

\( 3^{2k-1}=(ab-3^{2k-1}c'\,^3)+M\cdot (T/3) \)

Así que lo que puedes afirmar es que:

\( 3^{2k-1}\equiv (ab-3^{2k-1}c'\,^3)\quad \mod\quad M \) (pero no módulo \( q \)).

Saludos.